LSAT and Law School Admissions Forum

Get expert LSAT preparation and law school admissions advice from PowerScore Test Preparation.

User avatar
 Dave Killoran
PowerScore Staff
  • PowerScore Staff
  • Posts: 5852
  • Joined: Mar 25, 2011
|
#27053
Complete Question Explanation
(The complete setup for this game can be found here: lsat/viewtopic.php?t=3418)

The correct answer choice is (E)

The question stem asks you to identify a pair of variables that, when selected, allows for only one grouping of people to be selected. Answer choice (E) is correct because when M and Q are selected then L is not selected, and when L is not selected then N is not selected. From the second rule, then, P must be selected. And because Q has been selected, from the last rule K must be selected. Thus, when M and Q are selected, only the group of M, Q, P, and K can be selected.

Note that the work done in question #12 can be used to eliminate answer choice (C) because our hypothetical in #12 shows three different possible solutions when L and N are selected. In that same vein, the work from question #11 can be used to eliminate answer choice (D) because the selection of L and Q still allows for two possibilities (those possibilities are represented in the choice of N or P).
 lorein21
  • Posts: 17
  • Joined: Sep 30, 2011
|
#27535
Hi I was hoping someone could help me understand #13

I dont know how to do this problem. Would really appreciate the best tip on how to attack a question like this.


Thank You,

Lorein Abenhaim
User avatar
 Dave Killoran
PowerScore Staff
  • PowerScore Staff
  • Posts: 5852
  • Joined: Mar 25, 2011
|
#27537
Question #13: This is a Justify question, where you supply an answer that will produce the result requested in the question stem. In this case, the question stem asks you to identify a pair of variables that, when selected, allows for one and only one grouping of four people to be selected.

So, the correct answer in a problem like this must contain two variables that have some "power" in the game; that is, they affect other variables, either by knocking them out or forcing them to be selected. The more powerful the variable combination, the more likely it will be the correct answer. My first choice for a pair would be N and Q because from the last two rules they bring along L and K, completing the group. But, regrettably, that pairing was too obvious and the test makers did not include it among the five answer choices (still, understanding what the right answer could/should look like helps us understand what we are looking for).

Answer choice (E) is correct because when M and Q are selected then L is not selected, and when L is not selected then N is not selected. From the second rule, then, P must be selected. And because Q has been selected, from the last rule K must be selected. Thus, when M and Q are selected, only the group of M, Q, P, and K can be selected.

Note that answer choice (D) can be eliminated by the work we did in question #11.

Please let me know if this helps. Thanks!
 lorein21
  • Posts: 17
  • Joined: Sep 30, 2011
|
#27538
Thanks for the help!
 rameday
  • Posts: 94
  • Joined: May 07, 2014
|
#15496
I had a bit of an issue with this game. I have three answers all correct, A, D & E.
I don't know why A & D are not correct. for A I have JNLM as the only solution, for D I have KNLQ.

A
 Luke Haqq
PowerScore Staff
  • PowerScore Staff
  • Posts: 742
  • Joined: Apr 26, 2012
|
#15498
Hi rameday,

Let's go through (A), (D), and (E) to see why (E) has to be correct. The question is about which pair determines the other two to be chosen. I set up the initial diagram like this (variables to the right of || are those not selected):

IN || OUT

J/K, N/P __, __ || J/K, N/P, __

In other words, one of J and K will always be chosen, but not both, and the same for N and P. With that diagram, we can fill in the options from (A), (D), and (E):

IN || OUT

(A) J, N/P, L, M || K, N/P, Q
(D) K, N/P, L, Q || J, N/P, M
(E) K, P, Q, M || J, N, L

As you can see, only in (E) are all the spots determined. In (A) and (D), we don't know whether N or P will be chosen.

Hope that helps!
 diwil
  • Posts: 9
  • Joined: May 19, 2019
|
#65040
Hello,
What is the best way to attack this question? Besides re-using previous work, my only option was to try each answer choice out. But then, I wasted a lot of time, since correct choice was the last option, letter "E".
 Rachael Wilkenfeld
PowerScore Staff
  • PowerScore Staff
  • Posts: 1358
  • Joined: Dec 15, 2011
|
#65064
Hi Diwil,

Good question. When we first see this, it looks like there's NO WAY to guess what two variables we would be looking for. But there's a way to think through this. We want to find two variables that necessitate OTHER variables. That would mean that by selecting these two, we'd be able to select two more, making our complete set of 4 diplomats.

The first thing I did is look for variables that were sufficient conditions. That would be Q and N. But that's not a choice. So where can we look next? Well, we can think about what would force K or L to be out. If K is out, Q is out. If L is out N is out. So because of that, we want to look for situations that force either of them out.

The only thing that would push K out would be if J is in. If J, then not Q. If not Q, then LM. But that only gives us 3 of our 4 selections. We still could have either N or P. So let's look to what would push L out. The only thing that would push L out is if M and Q were both in. If M and Q are both in, K is also in (because if Q than K). We also have to select P, because N cannot be selected (if N then L). So we can prephrase than MQ is a possible answer choice, and that's what we see in answer choice (E).

Hope that helps!
Rachael.
 diwil
  • Posts: 9
  • Joined: May 19, 2019
|
#65330
Thank you Rachael, Yes it helps. I think it would take me some time to think through this question anyway...
Starting by looking for the sufficent condition will be the first approach I'll try next time for questions like this (even though for this question we couldn't get to the correct answer this way).
Thanks!
Diana
 gagomez
  • Posts: 1
  • Joined: Mar 09, 2020
|
#74295
Hi,

I'm confused about why the combination KMGL is not acceptable? Since it's N :arrow: L (meaning N is the Sufficient and L is the Necessary, isn't it correct to say that L can be used without N but N cannot be used without L?

Get the most out of your LSAT Prep Plus subscription.

Analyze and track your performance with our Testing and Analytics Package.